Серийная комбинация пружин

Когда система пружинных масс соединена вертикально с двумя последовательно соединенными безмассовыми пружинами, жесткость которых равна к 1 и к 2 к блоку массы м мы знаем, что на обе пружины действуют одинаковые силы. Пусть эта сила при колебаниях будет Ф .

Когда мы вычисляем эффективную жесткость пружины к с , почему бы не сказать, что результирующая сила, действующая на систему, равна 2 Ф ?

Нахождение чистой силы, действующей на вышеуказанную систему:

Когда блок прикреплен, система достигает положения равновесия за счет перемещений Икс 1 и Икс 2 .

В состоянии равновесия: 2 Ф "=" м г (Где Ф представляет собой величину усилия пружины, первоначально воздействующей на каждую пружину)

Так, к 1 Икс 1 + к 2 Икс 2 "=" м г (уравнение 1)

Когда система тянется вниз, она совершает колебания, теперь:

Общее удлинение быть Икс

Удлинение весной 1 быть Икс 1 и удлинение весной 2 be Икс 2 .

Общая сила пружины "=" к 1 Икс 1 к 2 Икс 2 к 1 Икс 1 к 2 Икс 2

Суммарные силы, действующие на систему "=" к 1 Икс 1 к 2 Икс 2 к 1 Икс 1 к 2 Икс 2 + м г "=" м г к 1 Икс 1 к 2 Икс 2 + м г
(из уравнения 1)

Итак, общая сила "=" к 1 Икс 1 к 2 Икс 2 "=" Ф 1 + Ф 2 "=" 2 Ф (так как мы знаем, что обе силы равны)

Таким образом, результирующая сила, действующая на систему, равна 2 Ф

Способ, которым я рассчитал эффективную жесткость пружины:

Икс "=" Икс 1 + Икс 2

2 Ф / к с "=" Ф / к 1 + Ф / к 2

2 / к с "=" 1 / к 1 + 1 / к 2

Но это не правильное уравнение. Что не так в том, чтобы принять результирующую силу, действующую на систему, как 2 Ф .

Ответы (3)

Когда пружины (предполагаемые невесомыми) подвешиваются вверх ногами, их растяжение равно нулю. Так что на самом деле

м г "=" к 1 Икс + к 2 Икс
при добавлении массы в систему.

Вы написали, что результирующая сила

Ф р "=" Ф 1 + Ф 2
и пришел к выводу, что это должно быть 2 Ф как если бы сила обеих пружин была одинаковой.

Итак, что вы должны были сделать, это записать результирующее смещение

Икс "=" Икс 1 + Икс 2
так что
м г к с "=" м г к 1 + м г к 2
где к с - эффективная пружинная постоянная. Затем это даст
к с "=" ( 1 к 1 + 1 к 2 ) 1

Пружины не имеют массы, поэтому натяжение в любой точке любой пружины одинаково. Ф . Для двух отдельных пружин с удлинителями Икс 1 и Икс 2 , у нас есть#

Ф "=" к 1 Икс 1 "=" к 2 Икс 2

и если рассматривать их как одну пружину, они имеют эффективную жесткость пружины к такой, что

Ф "=" к ( Икс 1 + Икс 2 ) Ф "=" к ( Ф к 1 + Ф к 2 ) 1 к "=" 1 к 1 + 1 к 2

Обратите внимание, что если мы соединим две одинаковые пружины последовательно (т. к 1 "=" к 2 и Икс 1 "=" Икс 2 ), то жесткость шарнирной пружины составляет лишь половину от двух ее составляющих. Это потому, что он производит ту же силу Ф с удвоенным удлинением отдельных пружин, так как Икс "=" 2 Икс 1 .

Итак, первое условие для последовательной пружинной системы: сила пружины во всей системе одинакова, т. е. натяжение пружин одинаково.

поэтому
к 1 Икс 1 "=" к 2 Икс 2

к е ( Икс 1 + Икс 2 ) "=" к 1 Икс 1
где к е эквивалентная жесткость пружины

поэтому, если система вертикальна, пусть сила тяжести, мг, будет Ф

следовательно Икс 1 "=" Ф / к 1 , Икс 2 "=" Ф / к 2 и Икс 1 + Икс 2 "=" Ф / к е

Икс 1 + Икс 2 "=" Ф / к 1 + Ф / к 2 "=" Ф / к е

1 / к е "=" 1 / к 1 + 1 / к 2

Теперь давайте разберемся с вашим вопросом, поэтому я рекомендую вам еще раз взглянуть на диаграмму свободного тела. Напряжение во всей системе равно Ф

Если у вас возникли проблемы, вы также можете попробовать связать это с цепями электрического резистора. При последовательном соединении ток через резисторы одинаков, а при последовательном пружинном соединении напряжение везде одинаковое. И в случае параллельного соединения ток разделяется на ветви, но каждая ветвь имеет одинаковую разность потенциалов, аналогично, в системе с параллельными пружинами две пружины испытывают разное напряжение, но растягиваются/сжимаются одинаково.

Для дополнительной справки: у меня есть некоторые проблемы с пружинами в серии.

и

https://physics.stackexchange.com/questions/311111/why-springs-in-series-experience-equal-force#:~:text=In%20series%20circuits%3B%20the%20current,force%20must%20be %20%20

Можете ли вы приложить диаграмму свободного тела для приведенного выше случая, на которой показаны все силы, действующие на систему?